Paper 3, Section II, F

Groups, Rings and Modules | Part IB, 2009

Let SS be a multiplicatively closed subset of a ring RR, and let II be an ideal of RR which is maximal among ideals disjoint from SS. Show that II is prime.

If RR is an integral domain, explain briefly how one may construct a field FF together with an injective ring homomorphism RFR \rightarrow F.

Deduce that if RR is an arbitrary ring, II an ideal of RR, and SS a multiplicatively closed subset disjoint from II, then there exists a ring homomorphism f:RFf: R \rightarrow F, where FF is a field, such that f(x)=0f(x)=0 for all xIx \in I and f(y)0f(y) \neq 0 for all ySy \in S.

[You may assume that if TT is a multiplicatively closed subset of a ring, and 0T0 \notin T, then there exists an ideal which is maximal among ideals disjoint from TT.]

Typos? Please submit corrections to this page on GitHub.